general questions on weaken Forum

Prepare for the LSAT or discuss it with others in this forum.
Post Reply
fire_fried_rice

New
Posts: 23
Joined: Thu Jul 13, 2017 11:06 am

general questions on weaken

Post by fire_fried_rice » Sat Nov 11, 2017 8:59 am

When solving weaken questions, we are supposed to find the conclusion, support and see how the reasoning behind it can be broken down or small bits where the reasoning can fall. I had slight problem trying to understand this conceptually until I found a solution of my own.

For me, I first find the conclusion. Then, I say to myself "say no to the conclusion" and try to find an answer choice that fulfills or supports the "no" conclusion. For example, on preptest 4 section 1 question 4, the conclusion was that "people would be better off not taking anti-seasickness medications" . If I use my method, the "no" conclusion would be "people would be better off taking the medications" and I try to find the answer choice that supports the new conclusion, which would be D) "the seasickness symptoms of people who took anti-seasickness medication would have been more severe had they not taken the medication".

I just want to confirm if this is a valid method or is this only possible with past preptests.

User avatar
Deardevil

Bronze
Posts: 496
Joined: Sat Jun 04, 2016 11:00 pm

Re: general questions on weaken

Post by Deardevil » Sat Nov 11, 2017 10:23 am

Not quite. You do not deny the conclusion.
Instead, focus on how the conclusion does not follow from the SUPPORT.

Premise: More people who take medication report symptoms of seasickness than those who do not take it.
Conclusion: People are better off NOT taking medication.

(D) The seasickness symptoms of the people who took anti-seasickness medication
would have been more SEVERE had they not taken the medication.

This suggests that a bunch of people perhaps naturally get seasick, and taking medication mitigates some symptoms.
But it weakens the argument because medications definitely are useful, even if it only gets rid of .00001% of the symptoms.

But if you treat the conclusion as false in the first place, then it would make little sense if you plug this bit into the stimulus.

Post Reply

Return to “LSAT Prep and Discussion Forum”